Question

Please show all your work and reasonings! 1). Consider the following information for three stocks Price per share 12/31/2003 $65.75 Price per shareDividends per Total shares outstanding 150 60 125 Stock 3/31/2004 $48.25 $28.125 $150.25 share $0.00 $1.25 $2.00 $24.125 $112.00 a) Calculate the value-weighted, equal-weighted, and price weighted index weights of stocks A, B and C at 12/31/2003 b) Calculate the holding period return on stocks A, B, and C in the first quarter of 2004 c) Calculate the value-weighted, equal-weighted, and price-weighted index returns on a portfolio of stocks A, B, and C in the first quarter of 2004?

0 0
Add a comment Improve this question Transcribed image text
Answer #1

1 a) Value Weighted: In Value weighted, a weight is assigned to each company based on the company's value. Investors value a company based on its market capitalization, which is the price of its stock times the number of shares outstanding.

Stock Price Per Share

Shares Outstanding

Market Capitalization

Weights
A 65.75 150 65.75*150 = 9862.5 (9862.5/25310)* 100= 38.97%
B 24.125 60 24.125*60 = 1447.5 (1447.5/25310)* 100= 5.72%
C 112 125 112*125= 14000 (14000/25310)* 100= 55.31%

Equal Weighted: In equal weighted, weights of all companies are equally, regardless of size. Here, there are three companies in the stock universe, and therefore each gets a one- third weight in the index.   

Price Weighted: In Price Weighted, the weight of a component is calculated by dividing the price of a security by the sum of prices of all index components.

Stock Price Per Share Weights
A 65.75 ((65.75)/(65.75+24.125+112))*100 = 32.57%
B 24.125 ((24.125)/(65.75+24.125+112))*100 = 11.95%
C 112 ((112)/(65.75+24.125+112))*100 = 55.48%

1b) Holding Period Return is the change in value of an investment, asset or portfolio over a particular period.

  • HPR = (End Value - Initial Value) / Initial Value
  • Stock

    Price Per Share(in $)

    12/31/2003

    Price per Share(in $)

    3/31/2004

    Holding Period Return
    A 65.75 48.25 ((48.25-65.75)/65.75)* 100= -26.62%
    B 24.125 28.125 ((28.125-24.125)/24.125)* 100= 16.58%
    C 112 150.25 ((150.25-112)/112)* 100= 34.15%

1c) Value Weighted index return:

  • Total Market Capitalisation 0n 12/31/2003 is $(65.75*150 + 24.125*60 +112*125) = $25310
  • Total Market Capitalisation 0n 03/31/2004 is $(48.25*150 + 28.125*60 +150.25*125) = $27706.25

Value Weighted Index return = ((Total Market Cap on 03/31/2004- Total Market Cap on 12/31/2003)/Total Market Cap on 12/31/2003)*100 = ((27706.25-25310)/25310)*100 = 9.467%

Equal Weighted index return: The return of an equal weighted index over a given period is calculated as a simple average of the returns of the index stocks

=( -26.62%+16.58%+34.15%)/3 = 8.03%

Price Weighted index return:

  • The price weighted index as of 12/31/2003 is (65.75+ 24.125+112)/3 = 201.875
  • The price weighted index as of 03/31/2004 is $(48.25 + 28.125 +150.25) = 226.625

The price weighted return =( (226.625/201.875)-1)*100 = 12.26%

Add a comment
Know the answer?
Add Answer to:
Please show all your work and reasonings! 1). Consider the following information for three stocks Price...
Your Answer:

Post as a guest

Your Name:

What's your source?

Earn Coins

Coins can be redeemed for fabulous gifts.

Not the answer you're looking for? Ask your own homework help question. Our experts will answer your question WITHIN MINUTES for Free.
Similar Homework Help Questions
  • Please show all work and show formulas! 1). Consider the following information for three stocks Price...

    Please show all work and show formulas! 1). Consider the following information for three stocks Price per share 12/31/2003 $65.75 Price per shareDividends per Total shares outstanding 150 60 125 Stock 3/31/2004 $48.25 $28.125 $150.25 share $0.00 $1.25 $2.00 $24.125 $112.00 a) Calculate the value-weighted, equal-weighted, and price weighted index weights of stocks A, B and C at 12/31/2003 b) Calculate the holding period return on stocks A, B, and C in the first quarter of 2004 c) Calculate the...

  • Please include all work and formulas 1). Consider the following information for three stocks Price per...

    Please include all work and formulas 1). Consider the following information for three stocks Price per share 12/31/2003 $65.75 Price per shareDividends per Total shares outstanding 150 60 125 Stock 3/31/2004 $48.25 $28.125 $150.25 share $0.00 $1.25 $2.00 $24.125 $112.00 a) Calculate the value-weighted, equal-weighted, and price weighted index weights of stocks A, B and C at 12/31/2003 b) Calculate the holding period return on stocks A, B, and C in the first quarter of 2004 c) Calculate the value-weighted,...

  • Please include work and formulas 2). Consider the following information for three stocks Time -0 Time...

    Please include work and formulas 2). Consider the following information for three stocks Time -0 Time -1 Total shares outstanding 100 200 200 Number of shares outstanding for B and C changed due to reverse and regular stock splits, Price per share S90 S50 $100 Price per share S108 $90 $55 Total shares outstanding 100 100* 400* Stock respectively a) Calculate the value-weighted, equal-weighted, and price weighted index weights of stocks A, B and C at time 0 b) Calculate...

  • 2). Consider the following information for three stocks. Time-0 Time= 1 Price per share $90 $50...

    2). Consider the following information for three stocks. Time-0 Time= 1 Price per share $90 $50 $100 Total shares Price er share $108 $90 $55 Total shares outstanding 100 100* 400* Stock outstandingp 100 200 200 "Number of shares outstanding for B and C changed due to reverse and regular stock splits, respectively a) Calculate the value-weighted, equal-weighted, and price weighted index weights of stocks A, B and C at time 0. b) Calculate the holding period return on stocks...

  • Consider the three stocks in the following table. Pt represents price at time t,

    Consider the three stocks in the following table. Pt represents price at time t, and Qt represents shares outstanding at time t. Stock C splits two-for-one in the last period.a) Calculate the rate of return on a price-weighted index of the three stocks for the first period (from t=0 to t=1). b) What must happen to the divisor for the price-weighted index in year 2? c) Calculate the rate of return of the price-weighted index for the second period (from t =...

  • 9. Consider the three stocks in the following table. P, represents price at time t, and...

    9. Consider the three stocks in the following table. P, represents price at time t, and Q, represents shares outstanding at time t. Stock C splits two-for-one in the last period. (LO 2-2) Pt 95 45 110 100 200 200 100 200 400 95 90 50 100 100 200 200 45 a. Calculate the rate of return on a price-weighted index of the three stocks for the first period (i 0 to t 1). b. What must happen to the...

  • 31-Dec-03 Price Stock 150.00 $50.00 Stocks W and X had 2 for 1 splits after the...

    31-Dec-03 Price Stock 150.00 $50.00 Stocks W and X had 2 for 1 splits after the close on Dec 31, 2003. Caloulate the geometric mean return from 2003-2004 for an unwelighted index. Hint: You can account for W and X's stock spit in the HPR equation.) O-1568 O.1925 2791 -0625 6.25 points QUESTION 15 Use the following information to answer the next two questions. Suppose that you create a price weighted index consisting of the following three stocks: Price 1/2/2019...

  • Consider the three stocks in the following table. Pt represents price at time t, and Qt...

    Consider the three stocks in the following table. Pt represents price at time t, and Qt represents shares outstanding at time t. Stock C splits two-for-one in the last period. P0 Q0 P1 Q1 P2 Q2 A 100 100 105 100 105 100 B 60 200 55 200 55 200 C 120 200 130 200 65 400 a. Calculate the rate of return on a price-weighted index of the three stocks for the first period (t = 0 to t...

  • Consider the three stocks in the following table. Pt represents price at time t, and Qt...

    Consider the three stocks in the following table. Pt represents price at time t, and Qt represents shares outstanding at time t. Stock C splits two-for-one in the last period. P0 Q0 P1 Q1 P2 Q2 A 88 100 93 100 93 100 B 48 200 43 200 43 200 C 96 200 106 200 53 400 a. Calculate the rate of return on a price-weighted index of the three stocks for the first period (t = 0 to t...

  • Consider the three stocks in the following table. Pt represents price at time, and O+ represents...

    Consider the three stocks in the following table. Pt represents price at time, and O+ represents shares outstanding at time i Stock splits two-for-one in the last period. Po P2 lo 100 200 200 P1 101 51 122 01 100 101 02 100 200 400 200 51 C 112 200 61 a. Calculate the rate of return on a price-weighted index of the three stocks for the first period (1=0 to r= 1). (Do not round intermediate calculations. Round your...

ADVERTISEMENT
Free Homework Help App
Download From Google Play
Scan Your Homework
to Get Instant Free Answers
Need Online Homework Help?
Ask a Question
Get Answers For Free
Most questions answered within 3 hours.
ADVERTISEMENT
ADVERTISEMENT
ADVERTISEMENT